Difference between revisions of "1989 AIME Problems/Problem 7"
Pi is 3.14 (talk | contribs) (→Solution) |
Namelyorange (talk | contribs) (New Solution) |
||
Line 2: | Line 2: | ||
If the integer <math>k</math> is added to each of the numbers <math>36</math>, <math>300</math>, and <math>596</math>, one obtains the squares of three consecutive terms of an arithmetic series. Find <math>k</math>. | If the integer <math>k</math> is added to each of the numbers <math>36</math>, <math>300</math>, and <math>596</math>, one obtains the squares of three consecutive terms of an arithmetic series. Find <math>k</math>. | ||
− | == Solution == | + | == Solution 1== |
Call the terms of the [[arithmetic progression]] <math>a,\ a + d,\ a + 2d</math>, making their squares <math>a^2,\ a^2 + 2ad + d^2,\ a^2 + 4ad + 4d^2</math>. | Call the terms of the [[arithmetic progression]] <math>a,\ a + d,\ a + 2d</math>, making their squares <math>a^2,\ a^2 + 2ad + d^2,\ a^2 + 4ad + 4d^2</math>. | ||
Line 8: | Line 8: | ||
Subtracting the first equation from the second, we get <math>2d^2 = 32</math>, so <math>d = 4</math>. Substituting backwards yields that <math>a = 31</math> and <math>k = \boxed{925}</math>. | Subtracting the first equation from the second, we get <math>2d^2 = 32</math>, so <math>d = 4</math>. Substituting backwards yields that <math>a = 31</math> and <math>k = \boxed{925}</math>. | ||
+ | |||
+ | ==Solution 2 (Straighforward, but has big numbers)== | ||
+ | Since terms in an arithmetic progression have constant differences, | ||
+ | <cmath>\sqrt{300+k}-\sqrt{36+k}=\sqrt{596+k}-\sqrt{300+k}</cmath> | ||
+ | <cmath>\implies 2\sqrt{300+k} = \sqrt{596+k}+\sqrt{36+k}</cmath> | ||
+ | <cmath>\implies 4k+1200=596+k+36+k+2\sqrt{(596+k)(36+k)}</cmath> | ||
+ | <cmath>\implies 2k+568=2\sqrt{(596+k)(36+k)}</cmath> | ||
+ | <cmath>\implies k+284=\sqrt{(596+k)(36+k)}</cmath> | ||
+ | <cmath>\implies k^2+568k+80656=k^2+632k+21456</cmath> | ||
+ | <cmath>\implies 568k+80656=632k+21456</cmath> | ||
+ | <cmath>\implies 64k = 59200</cmath> | ||
+ | <cmath>\implies k = 925</cmath> | ||
== Video Solution by OmegaLearn == | == Video Solution by OmegaLearn == |
Revision as of 15:09, 2 June 2023
Contents
Problem
If the integer is added to each of the numbers , , and , one obtains the squares of three consecutive terms of an arithmetic series. Find .
Solution 1
Call the terms of the arithmetic progression , making their squares .
We know that and , and subtracting these two we get (1). Similarly, using and , subtraction yields (2).
Subtracting the first equation from the second, we get , so . Substituting backwards yields that and .
Solution 2 (Straighforward, but has big numbers)
Since terms in an arithmetic progression have constant differences,
Video Solution by OmegaLearn
https://youtu.be/qL0OOYZiaqA?t=251
~ pi_is_3.14
See also
1989 AIME (Problems • Answer Key • Resources) | ||
Preceded by Problem 6 |
Followed by Problem 8 | |
1 • 2 • 3 • 4 • 5 • 6 • 7 • 8 • 9 • 10 • 11 • 12 • 13 • 14 • 15 | ||
All AIME Problems and Solutions |
The problems on this page are copyrighted by the Mathematical Association of America's American Mathematics Competitions.